Você está na página 1de 73

Week 8, Part A, Question 6 - Details

Anonymous 2 weeks ago Question 6 If a type-Ia supernova with luminosity 1010L were to be detected in RD1, what would its apparent brightness be as observed from Earth? Express your answer in terms of the Solar constant b and round to two significant figures. Vote this post up 2 Vote this post down Loh Siu Yin 2 weeks ago I'm not getting this. Given b=L4D2L and DL=D0(1+z) let Lsn be the luminosity of the supernova and Lsun and bsun be the Solar luminosity and brightness respectively then
bsnbsun=LsnLsun(DsunDLsn)2

What am I getting wrong here? Vote this post up 0 Vote this post down Comments Daniel Grabianowski 2 weeks ago I'm doing the same thing, and also getting it wrong. I'm pretty sure that I'm doing Q5 right, but I am getting it wrong as well. Dr. Plesser corrected a mistake in Q4 earlier, so perhaps there is also a mistake in the autograder for this question. We'll see tomorrow, I guess. Either that, or these are some pretty complicated questions. Vote this post up 0 Vote this post down Mark Polak 2 weeks ago Like Dan, I'm getting both 5 and 6 wrong. They seem reasonably straightforward to me (especially Q5). Has anyone got these right yet? Vote this post up 0 Vote this post down
A comment was deleted.

Loh Siu Yin 2 weeks ago Thank you for your hint Prasad, x.x e -xx . It was a rounding error. I had to round down to get answer accepted by the autograde. I had previously rounded up to get x.y e -xx. Vote this post up 0 Vote this post down

+ Add New Comment S V Prasad (Student) 2 weeks ago The formula is correct. But I am not sure why are you dividing the Luminosity value with Sun's Luminosity. It should be something like this b / b_sun = ((L_given*L_Sun)/(4*pi*(D_0(1+z))^2))/b_sun. You should get x.xe-xx Vote this post up 0 Vote this post down Comments Oscar Orta (Student) 2 weeks ago S V: I am also stuck here, but in your formula at the end of it, you have again /b_sun.....you have it already in the right side .....just take it as an observation...... Vote this post up 0 Vote this post down
A comment was deleted.

S V Prasad (Student) 2 weeks ago Oscar the requirement is to describe the answer in solar constant and that is what is explained. The right hand side of the formula is correct. I already got the correct answer to this problem. Vote this post up 1 Vote this post down Ioanna 2 weeks ago Oscar what S V Prasad says is correct. Basically we find the brightness of this object (given its luminosity and distance). But then we want this brightness as multiple of b_sun, rather than in W/m^2. So we divide by b_sun. (doing it in one step, means dividing both sides of the equation. You can not just divide one side by a number). We want b= x b_sun ie b/b_sun =x we find the brightness the normal way, so say b = y W/m^2 Then b/b_sun = (y W/m^2)/ b_sun Vote this post up 2 Vote this post down Anonymous 2 weeks ago The formula by Prasad (4*pi*(D_0(1+z))^2), whether (D_0(1+z)) is the answer from Q3? I applied the Distance from Q3 but is not accepted by the autograde. Please share your thoughts. Vote this post up 0 Vote this post down S V Prasad (Student) 2 weeks ago

The formula for Brightness is b = L / (4*pi*D_L^2) ---> This is what we studied in our class. We know D_L = D_0(1+z) From Q3 we already got the value for D_0 the coordinate distance. We know the value for z =5.34 and L in terms of Sun's Luminosity. After you get the values divide the brightness by solar constant (that is what is requested) you are all set. Keep in mind you need to convert light years to meters. Hope this helps. Vote this post up 5 Vote this post down + Add New Comment Satish Pisharody (Student) 2 weeks ago Prasad, D_0 would be the answer to Q3 or Q4? Vote this post up 0 Vote this post down Comments S V Prasad (Student) 2 weeks ago For Q3 you are asked to find Co-ordinate distance so it is D_0. For Q4 it is the other distance you need to find out. Vote this post up 0 Vote this post down
A comment was deleted.

A comment was deleted.

+ Add New Comment


A post was deleted.

Ingrid Paulussen 2 weeks ago First I have to convert D_0 tot other units, right? Vote this post up 0 Vote this post down Comments Mateusz Wielgosz 2 weeks ago So D_0 should be in AU? Vote this post up 0 Vote this post down Ingrid Paulussen 2 weeks ago I think D_0 should be in meters. Vote this post up 1 Vote this post down

Mateusz Wielgosz 2 weeks ago But isn't L_sun given for Earth at 1AU? therefore in numerator you have just 10^10, since L_sun will be cleared out. Vote this post up 0 Vote this post down Ingrid Paulussen 2 weeks ago I agree, but D_0 as we calulated is in lightyears, Vote this post up 0 Vote this post down Ioanna 2 weeks ago L_sun is in W (Watts). We just call 3.83E+26 W = 1 L_sun. Luminosity does not depend on distance, it is an intrinsic property of an object: how much energy it is radiating per second ie J/s = W. Now brightness does depend on distance. For a given luminosity, an object closer is brighter, and on object further away is dimmer. Vote this post up 2 Vote this post down Mateusz Wielgosz 2 weeks ago OK let's do it step by step. Imagine we converted D_0 to AU then Q6 is: [ 10^10 L_sun / 4 pi (D_L)^2 ] / [ L_sun / 4 pi (D_sun)^2 ] = [ 10^10 / (D_L)^2 ] / [ 1 / (D_sun)^2 ] Since we agreed on AU, then D_sun is just 1: Q6 = [ 10^10 / (D_L)^2 ] = [ 10^10 / (D_0)^2 * (z+1)^2] Right? Vote this post up 8 Vote this post down Ioanna 2 weeks ago Yes, that also works. But this gives the answer you have to truncate, as Gary mentioned. If you do it like S V Prasad said, where you just divide by the solar constant given in TOC, you do not need to truncate. Vote this post up 0 Vote this post down A. 2 weeks ago @ Mateusz Wielgosz

Right. Plain and straightforward ratios. The result should be in b units, so need to mess up with the exact value of that, namely the value of b Vote this post up 0 Vote this post down Frank Astier 1 week ago @Mateusz - best and simplest method. Vote this post up 0 Vote this post down Joo Romrio Fernandes Filho 3 days ago That's what I've been trying to do since the beginning, but my result is not accepted! I get x,y4849E-AB... Vote this post up 0 Vote this post down Joo Romrio Fernandes Filho 3 days ago Oh, I can't believe!!! All the time I was using the wrong distance!!! I was considering the past one insted of the "current"... Now I got it! Vote this post up 0 Vote this post down Joo Romrio Fernandes Filho 3 days ago Oh, I can't believe!!! All the time I was using the wrong distance!!! I was considering the past one instead of the "current"... Now I got it! Vote this post up 0 Vote this post down + Add New Comment stephen gould 2 weeks ago you just need to make sure the distance you are using in the denominator and numerator are in the same units. The suns distance is 1 au but it can be converted to km, m ly or anything else. The distance to the object was expressed in ly in earlier questions and can be converted similarly. Of course the more conversions you do the better chance you will get some rounding or input errors. And despite knowing this and agreeing with the above equations which do simplify I can't get the right answer. Reader beware. This is sort of a pain and a waste. B/bsun should = L/Lsun or 10 **10 times the ration of distances squared or so I think. The trick is the denominator distance D_L Vote this post up 0 Vote this post down Comments Sotiris Batsakis (Student) 2 weeks ago

Converting distance of question 3 from ly to AU. Also comments from Loh Siu Yin and Prasad above about rounding are helpful Vote this post up 0 Vote this post down Gary Lloyd-Rees 2 weeks ago Stephen, I used the same approach as you - don't forget to "redshift the distance" and you also need to truncate the answer.... Vote this post up 0 Vote this post down + Add New Comment Gary Lloyd-Rees 2 weeks ago In confirmation of others - to get the "correct" answer by the autograder you need to "truncate" to 2 sds instead of rounding. Wow the autograder is really given us a workout..... Edit to add - this applies if you are using the luminosity to brightness ratios approach from Week 4. Vote this post up 5 Vote this post down Comments
A comment was deleted.

A comment was deleted.

A. 2 weeks ago Thanks Gary, I got this Qu right at last.. i wish I could up vote your post more than once.. Vote this post up 0 Vote this post down Ingrid Paulussen 2 weeks ago and finally the truncating did the trick! Vote this post up 0 Vote this post down Juan F. Gonzlez Hernndez 2 weeks ago Truncating??? Vote this post up 1 Vote this post down Mateusz Wielgosz 2 weeks ago Instead of rounding as you should you just cut last digits.

Vote this post up 0 Vote this post down Ioanna 2 weeks ago I did not truncate, an my answer was accepted (I never round anything, other what I enter into the autograder). My 3rd figure is 4, so just rounding worked for me. If the calculations are different and you have a 5, then you would have to truncate. Vote this post up 0 Vote this post down Ioanna 2 weeks ago Ok, I see if one uses solar constant, no need to truncate. If one uses ratio of distances (basically recalculating b_sun, but cancelling some stuff) then one needs to truncate. Vote this post up 0 Vote this post down Gary Lloyd-Rees 2 weeks ago Ionna, if you use the straightforward ratios equation of L/Lo to b/bo route (with no interim rounding and using TOC conversions to AUs) then you get x.x5614...... which truncates to the "correct" answer of x.x Vote this post up 0 Vote this post down Ioanna 2 weeks ago Yes I tried that too, and indeed one needs to truncate in that case. I had done it same as S V Prasad, that was easier for me: find b. Divide by solar constant 1.37 10^3 W m^2 (this is rounded to 2 sig figs, so will slightly different result the L/Lo to b/bo route). The solar constant way gives x.y428ez, so I had just rounded normally. Vote this post up 0 Vote this post down + Add New Comment Juan F. Gonzlez Hernndez 2 weeks ago I calculated in this way: $$b/(b_S))=(L/L(sun))(1/(1+z)^2)(1A.U/D(Q4))^2$$ I got x.yze-wt and the bot says I am wrong...What do you mean by "truncate"? Please, answer if it is a bot issue too... Vote this post up -1 Vote this post down Comments A. 2 weeks ago

If you find something like (this is an example) 2.29e18...., truncation to two s.d. means using 2.2e18 instead of rounding to 2.3e18. Vote this post up 2 Vote this post down Juan F. Gonzlez Hernndez 2 weeks ago OK; truncating worked for me!!! Only Q5 remains... Vote this post up 0 Vote this post down + Add New Comment Juan F. Gonzlez Hernndez 2 weeks ago By the way, what is the order of magnitude you get? Just curious...I am desperated to finish this one since I know I am right with my numbers and I want to finisth the QB too, but not till I end this one... Crazy bot! Hahahaha... I know you... Can you hear me dear nasty bot? Hahaha... Just joking, ... Vote this post up 0 Vote this post down + Add New Comment Stoica Dorian - Bogdan 2 weeks ago

and we obtain the ratio distances in A.U., and before that, calculate angular distance should be known :) ) Vote this post up 8 Vote this post down Comments Wheeler Huneycutt 2 weeks ago

. My approach was to insert both using Slide 5, clip 2. (redshift is given, the

Stoica, you are the Man! Keep Calm and Love Physics!! I can not believe PoliSci majors at Dook can do this stuff! Vote this post up 1 Vote this post down Sohan P Jain 1 week ago Stocia, Just to confirm again, in your ratio, 10^10*(D_SUN^2/D_L^2), D_Sun is in A.U. and D_L is in L.Y. Is that right? (I'll convert to a common unit, prreferably to A.U.) Vote this post up 0 Vote this post down + Add New Comment

Stoica Dorian - Bogdan 2 weeks ago @ Juan : Wish i could give you the exact order of magnitude. However, is a negative number, bigger than -30, smaller than -19. Vote this post up 0 Vote this post down + Add New Comment Stuart Aitken 2 weeks ago Wow. This too me so long. I tried everything - every distance, every variety, etc, etc. Turns out I was dividing my final answer by the solar luminosity instead of the solar constant. Hah!!! Vote this post up 0 Vote this post down + Add New Comment Javier (Student) 1 week ago The autograder and me do not get along. I have tried the formula: Bsuper/Bsun= (Lsuper/Lsun)(1/(1+z)^2)(1 / Q4)^2, of course converting Q4(already OK) to AU,..I have truncated the Q4 formula, but also the autograder says no way,... I'm lost, Vote this post up 0 Vote this post down + Add New Comment Wheeler Huneycutt 1 week ago Javier, I think you need to move the 1+z inside the DA squared so the 1+z is to the fourth power, 1+z is squared if you using the D0 . Q4 was DA as the answer, no? HTH Vote this post up 0 Vote this post down Comments Javier (Student) 1 week ago Thanks so much Wheeler, I admit I have just done the mathematics and now I need to understand the physics,.. but definitively the auto grader and me are again friends. Thanks for your help!! Vote this post up 0 Vote this post down + Add New Comment Wheeler Huneycutt 1 week ago Javier, look at Clip 2 page 5 on the pdf . It gives DLD0DA relations as each multiplied by (1+z) and then squared. I leave it to you to explain to us a simple way to remember which is which. It is measurements of apparent luminosity, angular size and coordinate distance. Angle looks bigger than coordinate cause the light left early?

Luminosity drops one more z cause photons get stretched. Vote this post up 0 Vote this post down Comments Javier (Student) 1 week ago Wheeler, let me see if I've got it, Luminosity requires as shown by Clip 2 page 5 a redshift (1+z)^2, and then there is the relation between distances squared that add additional (1+z)^2,.. then that's why (1+z)^4. is that a correct deduction?.. for me it seems clear now, but I prefer double check Vote this post up 0 Vote this post down + Add New Comment Anna Czarina V. Cabe 1 week ago And I wondered why I could not ever get this right... I mistook a(t) for z. How brilliant. My two hours were for naught. Lesson learned.Always label your answers. Thanks everyone! Vote this post up 0 Vote this post down + Add New Comment Judit 1 week ago I'm utterly confused by all these distances and indices. I have no idea which one is which. Could anyone help me pls with sorting them out? I have distances in Q3, Q4, and a radius and angular radius in Q5. Which one do I need in the brightness formula? Thanks! Vote this post up 0 Vote this post down Comments Hy D Tran 1 week ago Judit, you want to look at clip 2, around slide 5. D0 is the distance today (now). The observed brightness (apparent brightness) is b from the equation on the upper right of the slide. Vote this post up 0 Vote this post down Anna Czarina V. Cabe 1 week ago Judit, Hy is right. Do not forget to multiply your answer from number 3 with the red shift (1+z)=this is D_L. Mateusz' solution from above should help. Here it goes: 10^10*(D_sun^2/D_L^2). Vote this post up 0 Vote this post down Judit 1 week ago

Thanks guys! I got the formula I just didn't understand what all the distances we discussed meant and which one should be plugged into the formula. For some reason this took me a lot of time to figure out. Vote this post up 0 Vote this post down + Add New Comment Wheeler Huneycutt 1 week ago This is really complicated. I am going to try to explain it.

D0 is where the thing is today. Distance from here to there today. DA=D01+z D_A is the distance we measure when we measure angular size of "bound" objects. DL=D0(1+z) D_L is the distance we measure when we measure Luminosity or brightness. Therefore: DL=DA(1+z)2
Now here's how to figure it out. Grab a relatively big galaxy in your hand and hold it about a foot away (thats about 30cm OK?). Hold it up where you see it real good about a foot away. Pretend that this is a while back in time, back when space was only half as big radially as it is today. Also pretend you are a big guy and can do this. Now notice the stars in this galaxy, some are near the edges of the galaxy. They are emitting photos, that is to say, the stars are shining. And some of the photons are aimed right at your eye. Pay close attention to the angular diameter of this galaxy, it should be about the angle between your eye and your thumb and forefinger. Now push the galaxy away to two feet (that's about 60cm). You just expanded space by double. Now imagine that that is where the galaxy is right now, but the light when it was only one foot away is the light reaching you now. D_0 is two feet. D_A is one foot. z is one. Space expanded by 1+1 or 2, double. (I want to say everything moved twice as far apart, but I know many will quibble with that.) Any instrument you have to measure the angular diameter is going to be measuring the photons emitted at one foot away. And it will measure the angular diameter today when the galaxy was one foot away. Now today the galaxy is two foot away. Now when you expanded space by double, think of it as your universe was one foot in radius of a sphere. And you made the sphere two foot radius. Well, when you expanded space, should you not have spread your thumb and forefinger apart by double, too? Well, yeah! you should. Try it with both hands and you'll see they just naturally separate by double, hold them at 90 degrees, that helps. Well why did the galaxy not spread apart? Because it was gravitationally bound together. It may look like a bunch of separate stars, but those stars are held together by gravity. And gravity is much stronger than any old expanding space. Well, space is not really expanding, what is really going on is the Big Bang blew everything to smithereens. Only the galaxies were able to hold everything together, thank goodness! If you can get a tennis ball, you can try this experiment using that as a sample galaxy, you can see how it holds together as space expands. Also, go get a handful of flour or dust or salt, and try pitching it about two feet up in the air. Neither Dust nor flour nor salt are gravitationally bound. This would be like measuring the angle between Arcturus and Betelgeuse, they are not bound so they separate when you expand space. Well, that's not true, but you know

what I mean. Things at great distance and great angular diameter spread apart. Well, it seems like a whole lot of bloviating just to explain the D_A. What about the D_L? It's even more complicated cause there's two things going on. First, we've got the same thing as measuring the angular size. Start out at one foot. You are holding a star or a supernova. We are going to measure the Luminosity which is a fine thing to measure, it is light itself. Well, actually, we are going to measure the brightness which spreads out as the light travels with the square of the distance. This we did way back when in week 2 maybe. It is sort of a like a sphere too, spreading out by area , distance squared. This part is easily taken care of like we did in week 2 with the formula b = L / 4 pi are squared, where are is D_L , the distance where the photons were emitted. Which would be D_A, EXCEPT, we have to modify two things. One, this star or supernova is firing photons at you. and they are redshifted, AND TWO they are delayed in frequency too! So you have to multiply D_A by (1+z) twice! Once for the number of photons observed, cause the source is moving away at (1+z) times speed, the bam, bam, bam frequency of photon after photon is decreased, that is seems slower as it moves away. Also, the very vibrational frequency, the color of the photon itself is lowered, due to the source moving away, this is the hum of the light that is too high pitched to hear, we can only see it as color or feel it as warmth, well, sort of anyway:) So 1+1=2 etc. Here's some "z" factors: http://en.wikipedia.org/wiki/List_of_the_most_distant_astronomical_objects It mentions M104, the Sombrero as being .004 the second galaxy redshift measured, next to Andromeda which has too much peculiar motion. Here's what SIMBAD says about M104 : z=.003642 http://simbad.u-strasbg.fr/simbad/sim-id?Ident=M104 Vote this post up 4 Vote this post down Comments
A comment was deleted.

Judit 1 week ago (edit) Thanks for the detailed explanation, Wheeler! So let me see if I got this right. I can either use D_now or D_A, correct? Could I just get D_L (the distance I need to use in the brightness formula) by multiplying the distance I calculated in Q3 (which is where the galaxy is now) by (1+z)? D_L = D_now*(1+z) where D_now is Q3, right? Should give the same result as D_A*(1+z)^2, where D_A would be Q4.

Vote this post up 0 Vote this post down Judit 1 week ago Apparently right, because my calculation was accepted by the grader. Wooohoo!!! For some reason this one was very hard for me to comprehend. Thanks again for the help! (P.S: I remember you from the hangout! :) ) Vote this post up 0 Vote this post down + Add New Comment John Owen 1 week ago What in heaven's name am I doing wrong ? I get D_L around e15 AU and an answer in the e-21 region (I've ? 3 right) I'm going to have a beer I blame it all on the person who, in Week 7, mentioned the pataphysical sciences, and I cannot get 'Joan was quizzical, studied pataphysical sciences at home' out of my head Vote this post up 0 Vote this post down Comments Siamak Kazemi 1 week ago Same here John. I'm stuck where you are. Vote this post up 0 Vote this post down Siamak Kazemi 1 week ago Hey John, I found out what was wrong. It's the Excel. Make sure you're dividing 10,000,000,000 by the denominator. You'll get an answer in the order of magnitude of e-22. That'll do it. Good luck. Vote this post up 0 Vote this post down Morin olivier 1 week ago Oooops John, i'm sorry, 'cause i think i'm the one, precisely. "Joan was quizzical/ studied pataphysical / Science in the home / Late nights all alone with a test tube / Oh, oh, oh, oh..." Vote this post up 0 Vote this post down + Add New Comment John Owen 1 week ago

Thanks, Siamak, you really got me to LOOK. Aaaaaaargh... I was using the value for a(t) instead of Z I must have had at least 20 tries,,,,,,,,,,,,,,,,,,,,,, On to 7 Vote this post up 0 Vote this post down Comments Siamak Kazemi 1 week ago You're welcome John. Vote this post up 0 Vote this post down + Add New Comment John Owen 1 week ago Morin You are forgiven you I can't stop singing 'Clerk-Maxwell's Silver Hammer' Regards John Vote this post up 0 Vote this post down + Add New Comment Anonymous 2 days ago Hi, I recall the formula given in the lecture (and we can redemonstrate/prove it by going back to the definition of Luminosity)
LL0=bb0D2, where D is in AU (Astronomical Unit)

So bb0=LL01D2 LL0 is given in the question and LL0=1010

So this is straightforward. But my question is: what is D ? This is the distance between the Supernova and the Earth, obviously. Is it the answer of the question 3 or the answer of the question 4 ? I guess this is the answer of the question 4 (including the redshift) but my answer is not accepted by the autograder. All is OK with units (distance everything in AU), I guess. I found something like this: x.yye-19. And you ? Where am I wrong, please ? Vote this post up 0 Vote this post down Comments

Sohan P Jain 2 days ago Hi, though I am not sure I can give you a practical advice - try both - that is why up to 100 tries are allowed. Vote this post up 0 Vote this post down Anna Czarina V. Cabe 2 days ago D is computed by multiplying D_0 to (1+z)... do not forget to square the whole expression. Hope this helps. Vote this post up 0 Vote this post down + Add New Comment MALINA ELEONORA COCIOCEANU 8 hours ago Please help with this one, I belive my conversion is wrong.So, we have b/b_sun=L/L_sun*1/D^2 D=1/(D_0*(1+z))^2. Vote this post up 0 Vote this post down + Add New Comment MALINA ELEONORA COCIOCEANU 7 hours ago Never mind, rounding error :) Vote this post up 0 Vote this post down Comments Dan Murphy 7 hours ago Glad you got it Vote this post up 1 Vote this post down

Week 8, Part A, Question 7 - Details


Anonymous 2 weeks ago Question 7 The luminosity of a type-Ia supernova declines by a factor of six from its maximum within about 40 days. How long will it take for the brightness of the supernova from Question 6 to decline by a factor of six from its maximal value? Express your answer in days and round to two significant figures. Vote this post up 0 Vote this post down Satish Pisharody (Student) 2 weeks ago Would this just be = number of days / (1+z)^2 ? So, 40/(1+z)^2 The logic being, brightness inversely scales to the square of the distance & the Supernova is receding by a factor (1+z) ? Any thoughts? Vote this post up 0 Vote this post down Comments Jeroen van der Graaf (Student) 2 weeks ago We are talking about an event that is very far away from us and happens on a very short timescale (in cosmological terms), so the dimming effect due to the distance increase in that time would be negligible. This question is about something that takes 40 days in a frame of reference comoving with the supernova. In our frame of reference the supernova is receding from us with relativistic velocity. What does special relativity have to say about time in such cases? Vote this post up 0 Vote this post down Mateusz Wielgosz 2 weeks ago I haven't sent my results to autograde yet, but there is no reason to square the (1+z) factor. We're not talking about brightness, just about time. So only elongation of period due to cosmological redshift should apply. That's first thing. Second thing if you divide 40 by 1+z you'll get shorter time, and that doesn't seem right. So I'd say it'll make more sense to multiply time by (1+z). However what bothers me is that with z = 5,34, we'll get time waaay longer than actual. But maybe that's just because that's how data for questions was chosen. Vote this post up 15 Vote this post down Sotiris Batsakis (Student) 2 weeks ago

Totally agree, not luminosity so the (1+z) factor must not be squared. Also time is dilated so answer must be a greater value than 40. Vote this post up 1 Vote this post down A. 2 weeks ago +1 Mateusz Wielgosz You are more than right IMHO Vote this post up 0 Vote this post down
A comment was deleted.

Satish Pisharody (Student) 2 weeks ago Thanks, Jeroen, Mateusz, A. for your inputs. But this is so counterintuitive... the Luminosity of an object, receding from us at relativistic velocities, falls to a sixth of its maximum luminosity in 40 days, but the apparent brightness takes much longer to reduce to 1/6th? I'm assuming this is because each succeeding photon has to travel a much longer distance to reach us & the information that the luminosity is falling also takes longer to get here the information gets stretched out. Can't see this happening to the tail lights of a car moving away from me, but I guess cars don't travel fast enough. May be I should change my intuition, as Ronen advised in one of the relativity video clips:) Vote this post up 0 Vote this post down Sohan P Jain 1 week ago Satish, I don't think the question says "falls to a sixth"; rather, it says "falls by a sixth" and therefore "falls to 5/6" of the maximum value. Another issue: if the luminosity falls by a sixth of the maximum in 40 days then it will become zero in just 240 days - too short on astronomical scale! Does it make sense? (Note: the maximum luminosity must be unique.) I am missing soimething here. Vote this post up 0 Vote this post down Satish Pisharody (Student) 1 week ago Sohan, the question does say "falls by a factor of 6", which to my mind means becomes 1/6th... however, this is not pertinent in answering the question. We are asked to find out the "time dilation" associated with the fall in luminosity as seen from Earth, which is the fall in "apparent brightness", b. Remember, Type I supernova is essentially a carbon star that goes out in one massive explosion, converting itself into Iron or whatever. Which may be the reason for the short luminosity (just days, a very short period in cosmic terms), not much of a star is left after the almighty blast, is my guess. Would like Justin or Dr.Plesser to comment on this.

Vote this post up 0 Vote this post down + Add New Comment Anonymous 2 weeks ago To any one out there, I approached this question as if it is asking me when do i see this dimming effect? To answer this it seems to require quite a few steps considering co motions and the like. Comments please. Vote this post up 0 Vote this post down + Add New Comment William Byars Samson 1 week ago The way I've approached it is to say that if an event takes 40 days and it has a redshift z, I am inclined to multiply that time interval by 1+z to give a longer time interval as special relativity would suggest. Unfortunately the auto-grader doesn't agree with my answer, so I'm stumped. Vote this post up 0 Vote this post down Comments Eric Dunbar 1 week ago Hmm. My logic is this. We get b=L/(4D2L) from an earlier problem. Since 4 is a constant and, at the time scale we're talking about, DL is effectively static we can simplify the equation to b=gL where g is a constant and g=(4D2L)1. Thus we know that bL so the brightness should diminish at the same rate as luminosity. We are told that luminosity decreases to a 1/6th over 40 days. That said, we need to remember that our here-and-now experience of how long this reduction in luminosity will take will be coloured (pun intended) by how long it took the light of the supernova to reach us. We learnt that the red-shift is a crude correction. Thus, William, I think you're completely on the right track. Perhaps you're not using the right adjustment with z. The correction ought to be quite simple. Vote this post up 0 Vote this post down Eric Dunbar 1 week ago One additional thought: your reasoning seems 100% correct to me. Are you perhaps missing brackets? Or using an incorrect z-value (5.34). At one point I had typed in the incorrect z-value, and, using logic that I knew had to be correct I couldn't solve it. One quick check of my values and I realised my mistake :). Vote this post up 1 Vote this post down

William Byars Samson 1 week ago You are absolutely right! For some reason I used z = 5.84 in this calculation so my answer wasn't close enough. Old age - failing eyesight - tiny fonts - AARRGGHH!!! Your sensible advice is much appreciated, Eric. Isn't it strange how we tend to look for something subtle when there's nothing subtle about this error. Vote this post up 0 Vote this post down Eric Dunbar 1 week ago That's the same mistaken z-value that I used. Vote this post up 1 Vote this post down + Add New Comment Anonymous 1 week ago William Byars Samson , I have put this whole redshift concept as answering the question: "An event that occurs THEN is what event that occurs NOW? You set up an equation accordingly, remembering that t=t(0)=1 at NOW. It seems that you are on the right track. Hope this helps Vote this post up 0 Vote this post down + Add New Comment Radek Krahl 1 week ago Using formula for time dilation:

t=t/(1v2/c2)
for obtaining v causing time dilation calculated from cosmological redshift (1+z) (t == 40) and then Hubble formula:

v=H0D
i've obtained distance to SN in question equal to ~4Gpc (~1 Gpc smaller than distance calculated in Q3) . I know that this may be stupid question - but where's the difference coming from? Vote this post up 0 Vote this post down + Add New Comment Anonymous 1 week ago What's the logic behind multiplication with (1+z)? Why can't we use time dilation formula (t=t01v2/c2)? (Velocity could be found from 0=1+v/c1v/c=1+zvc=(1+z)21(1+z)2+1). Vote this post up 2 Vote this post down Comments

Troy Williams 1 week ago I believe the cosmological redshift takes into account relativistic effects. If you take a look at this:http://astronomy.swin.edu.au/cosmos/c/cosmological+redshift I would be interested to find out if both methods agree. Vote this post up 0 Vote this post down Sohan P Jain 1 week ago What I found that the solution is independent of the information "factor of six" -- it could be six or six hundred or six million! I don't know why it is so. I want to hear from my coursera class-fellows. Vote this post up 0 Vote this post down Anonymous 1 week ago Sohan, You probably found that it is a function of time. As so an entity in the cosmos experiences an event (like luminosity dimming) and as time passes as such, 40 days. It will happen regardless of anything we do. The only consequence that we can realize is a change in apparent brightness. That is what we see, however, the only question that we answer is "when in time to we see it"? Vote this post up 0 Vote this post down Anonymous 1 week ago Sohan, typo "when in time do we see it" Vote this post up 0 Vote this post down Sohan P Jain 1 week ago Yes, that is what I found. The solution depends on "40 days" but not on "factor of six". I don't know exactly why it is so. Vote this post up 0 Vote this post down Anonymous 1 week ago Sohan, Without spending the time I, myself, need to fully understand this I am thinking about the redshift parameter z (how far back we are looking in time) is related to the scaling factor a(t) and a is related to t. The larger the z value, the further back in time we are detecting. We see that the radiation density was greater at z > 3300 (as said in the lectures) and the time past since the BB could approx 2,500< t <55000 years, as a rough guess. Again all these parameters are a relation

to time on a cosmological scale. These distances of objects (galaxies, clusters, etc.) from us are on such a massive scale (Mpc and lyrs, etc.) that we quantify them in "time" to get there (lyrs). Equally, for us to observe or detect those cosmological events, it is a matter of time. I do not pretend to know all but you can ask yourself this question "what should it depend on if not on time"? I believe you will find the answer that will satisfy you. Chow Vote this post up 0 Vote this post down David Gerstl 1 week ago Sohan, The reason that "40 days" matters and not "factor of 6" is that we're interested only in the time dilation. It could be any event taking 40 days. Assume someone was standing on the surface of a planet at the same position/relative velocity with a flag that they raised for 40 days (in their local time). We'd get the same answer for "how many days is the flag raised (when observed from Earth with a telescope). Just happens that the supernova change is an event that is actually observable from earth. Vote this post up 0 Vote this post down + Add New Comment Anonymous 1 week ago I thought this was straightforward, but I've spent hours on this and tried rounding up and down, and am not getting a correct answer. The most immediate solution that came to mind is 40*(1+z)=40*6.34=253.6=2.5e3 I've tried numerous variation on this theme, and can't think of another way to get this. I feel rather silly. Vote this post up 2 Vote this post down Comments Anonymous 1 week ago Never mind. It's hilarious. Just posting it here was enough to figure out the error of my ways. I should have complained a lot earlier! Vote this post up 0 Vote this post down Oscar Orta (Student) 1 week ago if you found that your answer is 253.6 and you try to tell the grader as 2.5e3 you are making a mistake.....2.5e3 = 2500 , not 250....check your arithmetic only.....just a simple mistake. Hope this helps. Vote this post up 4 Vote this post down + Add New Comment

Anonymous 1 week ago So I get this one ... sort of. Here's my problem: (1+z) = \lambda_observed / \lambda_emitted Now, most of this is due to relativity, but some should be doppler. Shouldn't we be removing that factor, or is it still small relative to the time dilation? Vote this post up 0 Vote this post down + Add New Comment Ignac Fetser 1 week ago Could someone please answer Anonymous' question? "What's the logic behind multiplication with (1+z)? Why can't we use time dilation formula (t=t0/(1v^2/c^2)^.5)? (Velocity could be found from..." The difference between the two methods is huge - the result with the time dilation formula is about 42 days while the t(1+z) is ~253 - why is that? Vote this post up 0 Vote this post down Comments Ignac Fetser 1 week ago ok i think i get it now - we would use the time dilation formula if the space were not expanding. but it does so that 40 days long light packet gets stretched out on its way here by the factor the universe has expanded between the time from its emission to observation.

Week 8, Part A, Question 8 - Details


Patty Allison 2 weeks ago Question 8 What was the temperature of the Cosmic Microwave Background when the supernova exploded in RD1? Express your answer in K and round to three significant figures. Vote this post up 0 Vote this post down Sotiris Batsakis (Student) 2 weeks ago Using formula at slide 2 page 5. T_obs is given at slide 6 page 4 Vote this post up 15 Vote this post down Comments A. 2 weeks ago This is kind of you, ! :-) Clip 6 is very enlightening.. Vote this post up 0 Vote this post down Anonymous 2 weeks ago Formula at slide 2 page 5? T_obs at slide 6 page 4? What videos are referred to please? Vote this post up 0 Vote this post down A. 2 weeks ago 2 and 6 Vote this post up 0 Vote this post down Anonymous 2 weeks ago C'mon do a bit of work Anon..... "Slide" = Video.... Vote this post up 3 Vote this post down Dave Fattori 1 week ago What has the formula in slide 2 clip 5 to do with the question? It doesn't have T in it. Vote this post up 0 Vote this post down Enith Paternina 3 days ago

T=2.726(1+z) Vote this post up 7 Vote this post down + Add New Comment Mateusz Wielgosz 2 weeks ago OK I just did this one, and I can say: for current CMB temperature use 2,725K, not 3K, because you won't get the right result. Vote this post up 3 Vote this post down Comments A. 2 weeks ago you probably mean T=2.726 K Vote this post up 1 Vote this post down Gary Lloyd-Rees 2 weeks ago For the non-continental Europeans, FYI, Mateusz's comma is a decimal point..... Vote this post up 5 Vote this post down Gary Lloyd-Rees 2 weeks ago Hi , still waiting for a resolution on Q5 - are you ready to tackle Part B? Vote this post up 0 Vote this post down A. 2 weeks ago Hi Gary, not yet. I can leave it for another day. I ain't got no time fighting with the typoed autograder... Vote this post up 0 Vote this post down William Gunn 2 weeks ago Apart from messing up my units after Q1 & Q2 (and the wierd thing on Q6) the only remaining part A question I'm struggling with is Q5 Bill Vote this post up 0 Vote this post down A. 2 weeks ago You shouldn't. Its straightforward once you use there a redzhifted distance..

Vote this post up 0 Vote this post down William Gunn 2 weeks ago Yeah - I don't understand it - mind you - I've had a lot of distractions with "real work" last couple of days - probably done something stupid (again) !!! Bill Vote this post up 0 Vote this post down A. 2 weeks ago Maybe this is a bit off topic but here it is: small angle formula. Find that angle in arcsec. Nominator: radius of galaxy (in whatever). Denominator: our distance (in whatever) from the galaxy when the light was emitted (reminds of a previous Qu, right?). Whatevers cancel out. Oh, don't you forget to multiply all that with our good ol' 206265 cuz you want this in arcsec, not radians. This post will self-destruct in 5''. Good luck Bill. Vote this post up 0 Vote this post down William Gunn 2 weeks ago LOL, thanks, but been there, done that! Part of my problem was thinking "this is one of the simplest Qs" - I can't believe it though - I have put all the questions for all the weeks into a VB program - and - I copy each question text as a comment - then I have to (marginally) edit the pasted question lines (just so they fit properly) Errrrrrr........... well yes.... typo Bill, typo. If I had reread the question text rather than relying on my pasted/edited copy..... oh well.... Ronen's not the only one who is a "spent force on this course"... ROFL. Oh well, gonna be a busy day at work today - but I must try to get a few more of part B done, would be nice to tie this all up before the weekend! Cheers, Bill

Week 8, Part B, Question 1 - Details


No tags yet. + Add Tag A. 2 weeks ago The ionization energy of a Hydrogen atom is Eion=2.17896e18J. Find the temperature T at which Eion=kBT. Express your answer in K and round to two significant figures. This should not cause us any problem.. Vote this post up 0 Vote this post down Ioanna 2 weeks ago There is also this : https://class.coursera.org/introastro-2012-001/forum/thread?thread_id=2214 Since noone has posted there, seems noone has a problem with it so far... Vote this post up 0 Vote this post down Comments A. 2 weeks ago I hadn't noticed that Ioanna. I am deleting this one....

Week 8, Part B, Question 2, Details


Sort replies by: Oldest first Newest first Most popular No tags yet. + Add Tag Anonymous 2 weeks ago Question 2 In fact, Hydrogen recombination is considered to have occurred at z=1100. What was the temperature of the Cosmic Microwave Background at this time? Express your answer in K and round to two significant figures. Vote this post up 0 Vote this post down Satish Pisharody (Student) 2 weeks ago Is the answer to this = Ans to Q1/(1+z) ? Need some conceptual help on this. Vote this post up 1 Vote this post down Comments Daniel Scandiuzzi 1 day ago Actually you have to multiply (1+z) and Q1. I looks like this: Ans to Q1*(1+z) Vote this post up 0 Vote this post down + Add New Comment stephen gould 2 weeks ago thought it might be T*(1+Z) but of course this doesn't work as most of my efforts on this homework Vote this post up 0 Vote this post down + Add New Comment Gary Lloyd-Rees 2 weeks ago Same approach as HWA Q8 should work Vote this post up 1 Vote this post down Comments Dave Fattori 1 week ago It doesn't work for me. Vote this post up 0 Vote this post down

Patty Allison 1 week ago T_c=T_o x ( 1+z ) using the T from Q8 Vote this post up 3 Vote this post down Gonzalo Ignacio Anda Olivares 1 week ago Use T_o the actual temperature of CMB Vote this post up 0 Vote this post down + Add New Comment Patty Allison 2 weeks ago use the T from HWA Q8 Vote this post up 2 Vote this post down Comments Pascual Prez Cuenca 1 week ago I thought this was the easiest question of all, but at the end it is the only one I'm now totaly lost. I thought that the only thing to do was multiply the answer of q1 for 1101. The aotoograder say no all time. Can someone give me an idea of what I'm doing bad.? Vote this post up 0 Vote this post down
A comment was deleted.

Latifis Konstantinos 1 week ago Patty correctly says: use the T from HWA Q8. It's only a multiplication, but don't use the answer of q1. You can find this T in Clip 6, slide 4... Vote this post up 3 Vote this post down Pascual Prez Cuenca 1 week ago Thanks Latifis I'll do after watching the clip 6 once more. Vote this post up 0 Vote this post down Konstantinos Siettos 1 week ago Latifis you mean the type from 8A not the T Vote this post up 0 Vote this post down

+ Add New Comment sio (Student) 1 week ago Can't get this one. I got all the 8 other questions but this last one. I followed the Patty's post and used the T found in HWA Q8 and multiplied it by 1101 but still it is not good. What is the order of magnitude do you have ? Is that something x10^4? Vote this post up 0 Vote this post down Comments Patty Allison 1 week ago T_o = 2.7255 K. z= 1100 so following the formula, you get the answer. Rounding to 2 significate digits and I didn't even convert to x.ye3. Vote this post up 7 Vote this post down sio (Student) 1 week ago Arf, ok, got it now. How makes things more complex than they are... Thanks for the help Patty. Vote this post up 1 Vote this post down + Add New Comment stephen weir 4 days ago okay, totally lost, Patty, when you say HWA Q8 are you referring to WK8, part A, Q8 which dealt with a temperature of a supernovae in a galaxy far, far away. What does that have to do with Part B, Q2? Vote this post up 0 Vote this post down + Add New Comment Patty Allison 3 days ago when solving problem 8 of part a of this week, we have to use the CMB temperature which is 2.7255 K. At the time I wrote the first hint, I was trying to avoid coming right out and saying the temp. They both have the Cosmic Microwave Background.

Week 8, Part B, Question 4, Details


Sort replies by: Oldest first Newest first Most popular No tags yet. + Add Tag Anonymous 2 weeks ago Question 4 The wavelength of acoustic waves in the Cosmic Microwave background was given in clip 8 as 201 kpc at z=1100. Estimate the size of such a perturbation today, in Mpc. Round to two significant figures. Vote this post up 0 Vote this post down Anonymous 2 weeks ago Simple. Space expanded by some factor and the wavelength too. Vote this post up 0 Vote this post down Comments A. 2 weeks ago Requiring this size of perturbation in Mpc almost gives away what are we supposed to do with that 1+z in our hands ... Vote this post up 2 Vote this post down + Add New Comment Patty Allison 2 weeks ago Anybody have any help to offer on this one? Just need a starting point.. Vote this post up 0 Vote this post down Comments Ioanna 2 weeks ago Although it may not seem so at first look, it just about comparing sizes (length in this case) at different times. We are told how big something was in the past (at z=1100). (This something is a wavelength, that was 201 kpc long.) How big (long) is it today? (The universe has expanded, and so has this length). So you just calculate the scale factor, and then convert the length then to length now. Vote this post up 6 Vote this post down Patty Allison 2 weeks ago

Thanks! I was having a brain cramp! I will try it again in the morning. Vote this post up 0 Vote this post down Patty Allison 1 week ago Got this one, thanks Ioanna! Vote this post up 0 Vote this post down + Add New Comment Eric (Student) 1 week ago This one should be straightforward, but can't get the grader to accept my answer. We have z = 1100, L = 0.201 Mpc (same as 201 kpc) Shouldn't the answer in Mpc be L times the square of the inverse of the scaling factor ? I have x.y..E+5, where x+y = 6 Vote this post up 1 Vote this post down Comments Radek Krahl 1 week ago Eric - you're scaling linear (one dimensional) value - length. So you must use first power of scaling factor (only one dimension to scale). Vote this post up 2 Vote this post down + Add New Comment Eric (Student) 1 week ago Oops, you're right, my bad ! Thanks ! Vote this post up 0 Vote this post down + Add New Comment Morin olivier 1 week ago Dear classemates... Am I tired or silly? Scale factor shouldn't be 1/(1100+1)? and the wave that i have to multiply by this scale factor shouldn't be 0.201Mpc? Vote this post up 0 Vote this post down Comments

Hy D Tran 1 week ago Olivier, You have most of the concepts correct. One more thing to consider: The scale today is 1 (by definition). When the wavelength was 201 kpc, the scale was (1/(1100+1)). Therefore, what is the wavelength today? Vote this post up 0 Vote this post down Morin olivier 1 week ago Arfffff, thank you Hy D Tran...i've entered my answered as answer of Q5 I'm finally tired to be silly and silly to be tired like that. Vote this post up 0 Vote this post down Anonymous 5 days ago This does not make sense to me.... 201KPC = (1/(1100+1), ?=1, so it is just 201/(1/(1100+1)) right???

Week 8, Part B, Question 3, Clarification


No tags yet. + Add Tag Anonymous 2 weeks ago For photons should we use the radiation density in the lecture slides 4.765e-5 or does this involve both photons and neutrinos? Vote this post up 0 Vote this post down ronen plesserINSTRUCTOR 2 weeks ago Use that number as the energy density in photons. Neutrinos are an interesting issue. We are not sure about their masses, so do not know how relativistic they are - in particular, some species may be massive enough to be nonrelativistic at the neutrino background temperature of about 2K. Vote this post up 1 Vote this post down Comments Anonymous 2 weeks ago I used that number but things did not work well. I did the following: 1.Multiplied \Omega of radiation by the critical density given in the correction page s 9.47e-27 kg/m^3 2.Multiplied by (1+z)^4 to get the density at the time of interest 3. Multiplied by c^2 to have the density as J/m^3. 4. Computed the temperature at the specified z and then used KT to find the energy per photon. 5. Divided the energy density by the energy per photon I checked the computations several times but the grader did not accept my trails. Vote this post up 0 Vote this post down Ioanna 2 weeks ago I did that differently and it worked. Clip 4, slide 5: there is a formula for energy density of radiation, that depends only T (and we have T. Well, we calculate it for z=1400, since we need it.) and g for photons is g=2 (mentioned in the video). My answer with this method, was accepted. Vote this post up 2 Vote this post down Anonymous 2 weeks ago Yes you are right. What about the third number? It should be straightforward but the grader rejects. I got for the number of ionizing photons x.yze7, where x=(y+z)/2 Vote this post up 0 Vote this post down Ioanna 2 weeks ago

I also have a problem with the 3rd number. I get x.yze7 (so same power) but not x=(y+z)/2 (but close). I have two different answers, depending on what I use for T today (2.7K or 2.726K. This affects T at z=1400). Both are wrong though. My 2nd number remains correct using either T (only the last sig fig changes by 1, and both possibilities are accepted). So it can not be an issue with T. I made another thread (called 'details' since it is more detailed) and also included the question (so it is easier to see what we are talking about). https://class.coursera.org/introastro-2012-001/forum/thread?thread_id=2234 Vote this post up 1 Vote this post down Anonymous 2 weeks ago Thank you Ioanna for your immense contributions.

Week 8, Part B, Question 5, Details


Sort replies by: Oldest first Newest first Most popular No tags yet. + Add Tag Anonymous 2 weeks ago Question 5 Imagine that the entire baryonic mass of the universe were converted to radiation. Find the temperature of the resulting radiation field. This is an alternative way of thinking about Olbers's paradox. Express your answer in K and round to two significant figures. Vote this post up 2 Vote this post down Anonymous 2 weeks ago I used the omegas of baryonic matter and radiation in the lecture. The ratio is the ratio of temperatures to the fourth. Knowing the current temperature of radiation (2.726 K), we can get the temperature if baryonic mass were converted to radiation. Did not work! What is wrong with this? Vote this post up 2 Vote this post down Comments Ioanna 2 weeks ago Hmm.. It says "Find the temperature of the resulting radiation field". I interpreted that as find T of radiation field created by baryonic matter converted to radiation. So the only radiation is that from the baryonic matter becoming radiation. I did not take into account the existing CMB and its temperature, to try and average the temperatures. My answer was also wrong by the way. Vote this post up 1 Vote this post down + Add New Comment Anonymous 2 weeks ago I have no clue how to even start on this - any suggestions?! Vote this post up 0 Vote this post down + Add New Comment Nicholas 2 weeks ago Having trouble with this one - here's my method. Baryonic density today = 4.17e28kg/m3 Convert to energy using E=mc2 ... getting the radiation field (kg/m3m2/s2=(kgm2/s2)/m3=J/m3)

Use radiation density formula from Clip4,Slide5 and solve for T. (rho=gsigmaT4/(4c) implies

T4=(4crho/(gsigma))
Units seem to work out here: (J/m3m/sK4m2/W)=(K4J/s/W)=(K4) I use g=2 to solve, and take the fourth root, getting a K between 0 and 100. No luck getting a correct answer yet - should I be using a different approach or different formula for energy conversion or energy density, or a different value of today's baryonic density, or a different value of g? Vote this post up 13 Vote this post down Comments Ioanna 2 weeks ago Well, if even you can not get a correct answer here, maybe the autograder is indeed wrong.. :) I also get something between 1 and 100 K, which is not accepted. Vote this post up 0 Vote this post down
A comment was deleted.

Anonymous 2 weeks ago Easiest way - to multiply the one-fourth power of bar/rad to the CMB temperature. It will give the same answer as your method, if you correct the formula for energy density: =4cT4 (see course corrections page). But the grader do not accept this answer too. Vote this post up 5 Vote this post down stephen gould 2 weeks ago I thought I understood but not quite from slides omega rad 4.765e-5 and omega baryonic was .256 (Omega b/omega r) to exp(.25) times K today : K result between 0 and 100 but did not work. last one to go - help Vote this post up 0 Vote this post down Oscar Orta (Student) 2 weeks ago Nicholas: I do not know if you already saw this: but check your T^4 formula.....I saw a correction in the course corrections section.....and the grader works.!. Maybe you already know this, but it is send with the best intentions. Vote this post up 1 Vote this post down stephen gould 2 weeks ago Nicholas

Did you get this to work. I used your approach with the correct and incorrect energy density formuales and even then added the existing Temperature from radiation to the resulting amounts. None worked. I think the formulaes make sense in terms of units since they all result in reasonable temperature results. Not sure if I am making a math error, there is some scaling factor off , auto grader off but it says it is fixed. Also wanted to make sure someone had this right to eliminate the auto problem. Vote this post up 0 Vote this post down Oscar Orta (Student) 2 weeks ago After converting to energy with E=mc^2, and then using the corrected formula to get T^4, (and then T, of course), the result is accepted by the grader. Hope this helps. Vote this post up 0 Vote this post down William Gunn 2 weeks ago Ahhhh - thanks - I've been distracted today - I'll check the corrections.... ty Bill Vote this post up 0 Vote this post down + Add New Comment
A post was deleted.

Shloka Ananthanarayanan 2 weeks ago The autograder seems to be fixed now! Vote this post up 0 Vote this post down + Add New Comment Juan F. Gonzlez Hernndez 2 weeks ago About this question and the cosmic destiny. Baryon burning to give "new life" and "heat" the universe with new radiation can not last forever. :( The amount of heat released is not well enough to make the Universe live forever! :( Vote this post up 0 Vote this post down + Add New Comment Wong Wo Chai 2 weeks ago E=mc^2=sigma*T^4/(2c), even I've used the corrected data for density, answer not accepted. Have I missed something? Vote this post up 1 Vote this post down Comments

Ioanna 2 weeks ago Look at the course corrections for the energy density of radiation (the formula is corrected there) Vote this post up 3 Vote this post down Wong Wo Chai 1 week ago I got it, tks loanna Vote this post up 0 Vote this post down + Add New Comment Richard Torson 1 week ago Look to course corrections......correct the density using E=mc^2....m=baryonic density(make sure to use c in meters...equate this to the corrected formula for pressure=(g*2*sigma*T^4)/c...use the value for a given in the course corrections to get a value for (g*2*sigma)/c....take the fourth root of this number to determine the answer....no need for the T_cmb.....we are just looking for the temperature resulting from the radiation. Vote this post up 2 Vote this post down + Add New Comment Eric (Student) 1 week ago Even though I went through all the hints above, I seem to be missing something still. I know how to extract the temperature from the energy density e=4T4/c in [J/m3], obviously. However, I have seen an equation above relating E=mc^2 [J] to e, which are not dimensionally consistent. Also, what is the mass used in E ? From these pieces of information, I cannot calculate e. I could easily get e from the given baryonic density in [kg/m^3] and the number of J/kg of baryonic matter, but I can't find this number. I am obviously missing something very simple. I would appreciate any additional hint, thanks. Vote this post up 1 Vote this post down Comments Ioanna 1 week ago E=mc^2 can also convert mass density to energy density, ie energy density (J/m^3) is =c^2 where is density of baryons (which is given earlier in qu3 (or you can calculate it by _m x _c = _m,b

The value for _c (critical density) has a course correction, so do not use the value in the slides but the corrected one. Vote this post up 7 Vote this post down A. 1 week ago This is a brilliant explanation! Vote this post up 0 Vote this post down Richard Torson 1 week ago Nicolas did a great job with the dimensional analysis above....check it out, and as Ioanna stated above...she is the belle of the discussion forum!!! Vote this post up 2 Vote this post down Eric (Student) 1 week ago Of course, this is exactly what I missed : J/kg = (m/s)^2, so I should have guessed the c^2 factor ! Thank you Ioanna ! If there is a medal for the best helper, you should get it ! Vote this post up 5 Vote this post down + Add New Comment A. 1 week ago If E=g2T4c then T=(Ec2g)1/4 (eq.1), where E I call energy density in Jm3, is in K, c and from the TOC. BUT E=mc2E=EV=mVc2=c2E=c2 (eq.2), where is mass density in kgm3.

(eq.1)(eq.2)T=(c32g)1/4 (eq.3) kgm3m3s3Jsm2K41/4=[sm2K4kgm3J m3s3]1/4= =[kgm2K4s2J]1/4=[JK4J]1/4=[K4]1/4=K


Radiation means photons so for a gas of photons it is g=2, so:

(eq.3)T=(c34)1/4
and I find a T in [12K, 18K]. Am I on the right track? I'd rather ask all of you than the autograder. Vote this post up 8 Vote this post down Comments Ioanna 1 week ago

The method looks correct. I was about to say your previous result (e-2) did not look right. Should be e1. What you have now looks ok. Vote this post up 0 Vote this post down A. 1 week ago Ioanna, thanks for your feedback! I tried all this elaborate reasoning which probably looks correct and I previously entered c in km/s in Excel instead of m/swhich is the proper unit. A minor detail can keep you struggling. Vote this post up 2 Vote this post down
A comment was deleted.

A comment was deleted.

+ Add New Comment Sean Storey 1 week ago Thought this was going to be straight forward but cannot get the autograder to accept my answer. I'm calculating the energy density from the baryonic density as 8.511e-10 Does this look right ? The rest should be straight-forward e.g. mulitple by 1/a( Radiation constant from course corrections) then take the 4th root Vote this post up 0 Vote this post down + Add New Comment Ioanna 1 week ago That doesn't look right Sean (it is 1 order of magnitude off). I did your calculation backwards, and I think you have used the critical density (_c), rather than the baryonic density. The density of baryons is given in qu3 a), or you can calculate it more accurately by _b x _c (and look at course corrections for the correct value of _c). Vote this post up 0 Vote this post down Comments Sean Storey 1 week ago Thanks - that's it. Vote this post up 0 Vote this post down

+ Add New Comment Chiok Ching Chaw (Student) 1 week ago Hi. Got a related question. Radiation constant is given by a = 4*sigma/c. However substituting sigma = 5.670e-8 and c = 1.49597870691e11, I am not able to get a = 7.565767e-16 as stated in the Course Correction notes, am I missing something? Vote this post up 0 Vote this post down Comments ronen plesserINSTRUCTOR 1 week ago Where did that c come from? We have c = 2.998e8 m/s Vote this post up 2 Vote this post down Oscar Orta (Student) 1 week ago your measure of c is wrong.....it should be c= 2.997924 e 8 m/s...then you will get it right...I hope this helps. Vote this post up 0 Vote this post down Chiok Ching Chaw (Student) 1 week ago Opps! My bad! I got mixed up with the value for AU! Thanks! Vote this post up 0 Vote this post down + Add New Comment Anonymous 1 week ago I've tried this so many times, I can't see where I've gone wrong. I've used density = 4.17e-28 , c=2.99792458e8, sigma= 5.6703e-8 and I keep coming up with a figure of the order e-2. I've also tried using the calculated version of density which is very close to but just a bit smaller than the given density and get pretty much the same answer. Can anyone see my mistake? Vote this post up 0 Vote this post down Comments
A comment was deleted.

A comment was deleted.

A comment was deleted.

+ Add New Comment serge 13 hours ago

What expression for the radiation density (equation of state for phonons) shall we use now? Whether the erroneous one in the slides, or, the corrected one in the course corrections which differs by a factor 8? (that is whether the autograder has been adjusted to corrected equation or it remains with this mistake)? Vote this post up 0 Vote this post down Comments
A comment was deleted.

ronen plesserINSTRUCTOR 13 hours ago Use the corrected expression. Vote this post up 0 Vote this post down + Add New Comment Anonymous 40 minutes ago I think frustration is setting i and I am missing something. . Converting density into E_p: Baryonic Density * (2.9979e8)^2 E_p = (2*g*sigma*T^4)/c; Solving for T^4 = (E_p*c)/(2*g*sigma) since g = 2 then T^4 = (E_p*c)/(4*sigma); since (4*sigma) /c = 7.565767e-16 then the inverse = 1.32174305e15 which is equal to c/(4*sigma); T= (Baryonic Density * (2.9979e8)^2 * 1.32174305e15)^.25 It seems my power is correct e^1 but my coefficent is incorrect. What am I missing? Thank you

Week 8, Part B, Question 6, Details


Sort replies by: Oldest first Newest first Most popular 1 2 Next No tags yet. + Add Tag Anonymous 2 weeks ago Question 6 In this set of linked problems we will take a look at exponential expansion - which we think describes both the distant future of our universe and an important phase in its very early past. In exponential expansion with a(t)=eH0(tt0) the coordinate distance - the distance now between the position of a photon at time t1 and its position at time t2 is D=cH01(eH0(t2)eH0t1). Consider a photon observed at t0. Find the distance at which it was emitted and write this in terms of the observed redshift z. Your answer should be an expression containing z, H0, and c. Vote this post up 0 Vote this post down Juan F. Gonzlez Hernndez 2 weeks ago A necessary clarification in the statement of the problem: I can calculate the proper distance integrating the scale factor inverted in the following way:

d=ct2t1dta(t)
Inserting the scale factor a(t)=exp(H0(tt0)), we get

d=ct2t1dtexp(H0(tt0))=ct2t1exp(H0(t0t))dt=cH0(exp(H0(t0t2)exp(H0(t0t1)) Set t0=0 and we have the condition Ronen gave us in the problem. Calculus and integrals are
cool (and rocks)like bow-ties!!!! LOL Let me know if I am right or not...It is relevant in order to get the right expression relation c, H0 and the redshift z. Moreover, it is also relevant in the calculation of the "particle horizon". Vote this post up 3 Vote this post down Comments Anonymous 2 weeks ago I did the same, but integrated from t to t0. I obtained cH(eH0(t0t)1). The exponent is readily expressible in terms of z. What is the formula for the particle horizon? Vote this post up 0 Vote this post down + Add New Comment

A. 2 weeks ago You don't need calculus for that. It's straight plain n simple algebra. Vote this post up 2 Vote this post down + Add New Comment Juan F. Gonzlez Hernndez 2 weeks ago I know, but I was having fun with advanced topics in Cosmology I am studying now, hahaha... Vote this post up -6 Vote this post down + Add New Comment Alexander Edward Drew 2 weeks ago Is it correct to start by setting a(tem)=1/(1+z)=exp(H0(temt0)) and then solve for exp(H0tem) ? Using this approach and substituting a constant for H0t0 I reach an incorrect answer. Vote this post up 0 Vote this post down + Add New Comment
A post was deleted.

A. 2 weeks ago I assumed H0t00 and it worked miracles but am I justified assuming this? I am in a conceptual loss here, although the grader likes it and says it's OK. Anyone with a good insight about my assumption? Vote this post up 1 Vote this post down Comments Loh Siu Yin 2 weeks ago If z is high then t0 is large. i.e. we are in an old universe and eH0t0 is small. Vote this post up 0 Vote this post down
A comment was deleted.

A comment was deleted.

Ioanna 2 weeks ago With the new definition Ronen gives below, t0=0. So H0t0=0 Also a(t0)=eH0(t0t0)=eH00=1 (ie scale factor today is 1) And generally the scale factor at time t is a(t)=eH0t Vote this post up 0 Vote this post down + Add New Comment

Satish Pisharody (Student) 2 weeks ago Am stuck up on this problem & the succeeding linked problems. Appreciate help with the steps! Vote this post up 1 Vote this post down Comments
A comment was deleted.

+ Add New Comment ronen plesserINSTRUCTOR 2 weeks ago A few clarifying comments on this one: First, in going from the first expression to the second I have tacitly taken t0=0. In an exponential universe this is a natural way to measure time because there is no obvious big bang - singularity is at t. I could have been more clear on this. Second, in order for the signs to make sense as I wrote them you have to have t2<t1 (otherwise D is negative). To pick nits, I also have a completely unnecessary set of parentheses around t2. The particle horizon at time t1 (about which I did not ask) would include the most distant events from which a photon could have arrived by t1. To find this, fix t1 and adjust the emission time t2 to maximize D. The result is interesting. The event horizon at time t2 is the maximal distance from which a photon emitted at t2 will ever reach us. To find this you fix t2 and adjust t1 (when in the future we will receive the photon) to maximize D. Vote this post up 10 Vote this post down + Add New Comment Satish Pisharody (Student) 2 weeks ago Thanks, Dr.Plesser. Back to the drawing board on this one to see if I can get back on track. The concept still eludes me, may be will become clear as I go through this set! Vote this post up 0 Vote this post down + Add New Comment Ioanna 2 weeks ago Thanks for the clarifications Ronen, but I am still confused with this. So we are saying t goes from t to t0=0. So then t is always -ve, right?. (Well if t0 is today, we have -ve t in the past, and +ve t in the future) So the a(t) formula gives us scale factor a(t)<1, for -ve t (so times earlier than t0). And a(t)>1 for +ve t, ie times later than t0 (ie in the future). But if t is -ve, and we already have a '-' in the exponents in the 2nd formula, the exponents are then +ve? (for times earlier than t0). But for -ve t (times earlier than t0) the scale factor is <1. And the +ve exponent will give us scale factor >1.

And is t2 is earlier than t1? You mention t2<t1 but it depends if these are +ve or -ve to find what is earlier. I have already answered the question 'correctly', but I am still not sure I understand the answer.. I basically guessed it, rather than find out what it is the proper way.. Vote this post up 0 Vote this post down Comments Ioanna 2 weeks ago Ok, further to my post above, I 'think', with -ve t's, we still need to keep the '-' in the exponents in the 2nd formula. We have that the scale factor is a(t)=eH0t. Then eH0t=1a(t), and everything seems to work ok. I think... Vote this post up 0 Vote this post down Alexander Edward Drew 2 weeks ago I think the fact that t can go back to negative infinity is only needed in this problem in so far as to let you know that we are taking t0=0 and that tem is some negative time prior to that that could be any magnitude. You can then replace tem in the distance equation in terms of z and t0=0 and solve for D in terms of z, c, and H0. Vote this post up 0 Vote this post down + Add New Comment A. 2 weeks ago These are the outskirts of calculus and we take, thanks to Mr. Ronen and this excellent question, a small idea of how useful this section of mathematics can be. But I know some basic n rusty algebra and it helped me through this course. I would produce a train equation series, as my friend Gary from Canada would call it, but I guess Prof. Plesser's comments above are more than detailed. Vote this post up 0 Vote this post down Comments Satish Pisharody (Student) 2 weeks ago Thanks, A. for all your contributions to this forum. You & Ioanna have made the forum a great place for after-class learning... I am yet to completely grasp the intricacies of week 8, and find myself struggling a bit. Hopefully the autograder will approve of my efforts when I get to it. Now for Q8 & Q9 & I should be done:) Vote this post up 3 Vote this post down A. 2 weeks ago

I am yet, if I ever, to completely grasp W8 myself, too... Vote this post up 0 Vote this post down + Add New Comment Carlos Sarno 2 weeks ago I have an answer which was accepted but I can't see why. The eqn seems to be appropriate for distance now (i.e as given in the question), not distance when emitted. I.e. I omitted 1/(1+z) from my original (unaccepted) answer and then it was accepted. This is the only question in the whole course where I've been puzzled!!! Vote this post up 1 Vote this post down + Add New Comment Brian Ritchie (Student) 1 week ago Well I'm very gradually becoming aware of what this question entails but I'm still far from solving it. This is what i ended up with, which is wrong. c/H_0/(1+z) - c/H_0 I must admit I'm having difficulty getting my head around what's going on here lol Vote this post up 0 Vote this post down Comments A. 1 week ago Start off by t0=0, as Prof. Plesser states above. Then it's H0t0=0, thus eH0t0=e0=1. Then, treat t2 as tand t1 as t0. Vote this post up 7 Vote this post down
A comment was deleted.

Eric Dunbar 1 week ago (delete & repost) Aaargh. I can't make heads nor tails yet of this question.

a(t)=eH0(tt0) Since t0=0 that means that 11+z=eH0(t) and 1+z=eH0(t) D=cH10(eH0(t2)eH0t1) Next, if we are to treat t2 as t and t1 as t0 then I should be able to simplify the above expression for D
Well, slap self on head again. I forgot that you have to explicitly tell the autograder where the multiplication sign appears. Remember those pesky *s. This question is doable. PS My son is now up. Good, solid three hour nap and now hopefully we'll have him down by 8 (three hours is pretty long for him nowadays so he might stretch things out to 20.30 ;-).

Vote this post up 7 Vote this post down + Add New Comment Brian Ritchie (Student) 1 week ago - thanks for that - I'll give it a go after another cup of coffee! Vote this post up 0 Vote this post down + Add New Comment Brian Ritchie (Student) 1 week ago Hmmm - that gives me the same equation. Looks like there is something wrong with the way I'm introducing z? (exp)-H_0t = 1/1+z ??? Vote this post up 0 Vote this post down Comments Carlos Sarno 1 week ago exp(-H0*t) = 1+z not 1/(1+z) Vote this post up 2 Vote this post down + Add New Comment A. 1 week ago Looks that that coffee was not so strong. It is exp(H_0*t) = 1/(1+z) or the same in Latex eH0t=11+z. Be careful with the math formality. Do not forget those * and where to put brackets. I ended up on the conclusion that H_0*t_0 should be 0, namely t_0 should be 0, otherwise I couldn't have a formula with the symbols/quantities required. So, I said, let t_0 be zero, then worked some algebra, then I found an expression which looked to simple to be true and correct. I asked the grader and it replied it was OK. Vote this post up 3 Vote this post down Comments
A comment was deleted.

Ioanna 1 week ago , I think your fraction should be inverted? We have:

a(t)=11+z And a(t)=eH0t Then eH0t=1a(t) So eH0t=1+z


Vote this post up 7 Vote this post down

A. 1 week ago Right on, Ioanna, I just corrected it. Vote this post up 0 Vote this post down + Add New Comment A. 1 week ago ie 1/1+z is 11+z whereas 1/(1+z) is 11+z, namely, much different expressions. Vote this post up 0 Vote this post down + Add New Comment Anonymous 1 week ago Hey Stavros, My expression in the parentheses is 1/(1+z)-1, yet the answer is wrong. Any Ideas? Thanks Vote this post up 0 Vote this post down Comments A. 1 week ago Yep. a(t)a(t0)=11+za(t)1=a(t)=11+za(t)=eH0(tt0)=eH0teH0t0=eH0t1=11+z

eH0t=11+zeH0t=1+z
Vote this post up 5 Vote this post down Carlos Sarno 1 week ago Also obtained simply from Ronen's a(t) eqn directly by setting t0=0 and inverting. Vote this post up 1 Vote this post down Colin Derek Dando 1 week ago Carlos, someone else from Colchester, UK Vote this post up 0 Vote this post down Anonymous 5 days ago Inside my brackets I get (1+z)-(1+z)... how do i differentiate between t1 and t2??? Vote this post up 0 Vote this post down

Carlos Sarno 1 day ago Hi Colin. Only just seen your post. Moved on to other courses once I finished this one. I'm actually in Colne Engaine near Halstead. Vote this post up 0 Vote this post down + Add New Comment Brian Ritchie (Student) 1 week ago Yes, I understand that (I missed out the brackets in my post above). When I review it it looks OK but the grader is not accepting my answer. Vote this post up 0 Vote this post down + Add New Comment Paul Robinson 1 week ago I spent hours on this yesterday, looking back at my notes I came very very close to correct. After a double espresso and reviewing comments from A. I stumbled upon the right answer - and couldn't believe how simple it was until the autograder gave me a wink back ;-) Thank you again A. Vote this post up 0 Vote this post down + Add New Comment Brian Ritchie (Student) 1 week ago Ah, yes - thanks Stavros - one extra coffee did it :) - the minus sign oh dear. lol Vote this post up 0 Vote this post down + Add New Comment A. 1 week ago With all that coffee that we all drank during this course, I hope our stomachs will get better after it finishes. If this course ever assumes a sponsor, it would probably be a coffee corporation. Vote this post up 4 Vote this post down + Add New Comment Stoica Dorian - Bogdan 1 week ago I still don't get it...why don't you use Clip 2 Slide 4???? Vote this post up 3 Vote this post down Comments

A. 1 week ago That's probably because we like drinking coffee, I guess...:-)) Vote this post up 0 Vote this post down Nancy Nelson 1 week ago And getting only 6 hrs of sleep over a span of 2 days does not help either...but that is probably why I had to drink so much coffee... Vote this post up 0 Vote this post down Eugene 5 days ago I also like drinking coffee, but it doesn't help now! Vote this post up 0 Vote this post down + Add New Comment Kristina Mois 1 week ago Enough coffee, I just poured me some wine to celebrate my completion of the final problem set! Thank you for your help, Ionna, and many others who have been contributing here. You saved me a lot of headaches on this problem set. You banged your heads against the wall with autograder issues and conceptual issues, so that others like me could waltz right through after you. Vote this post up 3 Vote this post down Comments A. 1 week ago I tried to bang my head less with the autograder cuz my time available for this kind of adventure is limited :-) Nonetheless, this kind of struggle with the autograder shows that there are students here on this course who are quite energetic regarding the homework that are given and do not take it in a passive attitude. I think this is great and all of us owe them a lot. I am sure this attitude helped Prof. Plesser refine and better the material given for the rest of the students. Vote this post up 0 Vote this post down + Add New Comment Brian Ritchie (Student) 1 week ago Yes, same here - thanks to you all! I finished the final set yesterday. :) You were right Stavros, it just wouldn't have been right to get so close and not complete the course - would have bugged

me no end. Now I need to re-watch the videos at leisure - lots of interesting stuff which I never really absorbed. lol Vote this post up 0 Vote this post down Comments Sue Terwilliger 1 week ago I was going to settle for 90%, but you guys talked me into plugging away! I finished all but 2 questions yesterday, and I have 2 weeks to get them! Vote this post up 0 Vote this post down Paul Robinson 1 week ago Sue, go for 100%. Plenty of friendly assistance around, and believe me you will feel so great when you get there :-) Vote this post up 0 Vote this post down Sue Terwilliger 1 week ago Got it, and it didn't take the whole 2 weeks! Now I have to concentrate on Calculus! Vote this post up 0 Vote this post down + Add New Comment Eric Dunbar 1 week ago I'm going to take the liberty of combining Ronen's post with the question so I can see everything in the original, glorious Latex (I can't wrap my head around t1, t2 and t0). Ronen's clarification: A few clarifying comments on this one: First, in going from the first expression to the second I have tacitly taken t0=0. In an exponential universe this is a natural way to measure time because there is no obvious big bang - singularity is at t. I could have been more clear on this. Second, in order for the signs to make sense as I wrote them you have to have t2<t1 (otherwise D is negative). To pick nits, I also have a completely unnecessary set of parentheses around t2. The particle horizon at time t1 (about which I did not ask) would include the most distant events from which a photon could have arrived by t1. To find this, fix t1 and adjust the emission time t2 to maximize D. The result is interesting. The event horizon at time t2 is the maximal distance from which a photon emitted at t2 will ever reach us. To find this you fix t2 and adjust t1 (when in the future we will receive the photon) to maximize D. The question:

In this set of linked problems we will take a look at exponential expansion - which we think describes both the distant future of our universe and an important phase in its very early past. In exponential expansion with a(t)=eH0(tt0) the coordinate distance - the distance now between the position of a photon at time t1 and its position at time t2 D=cH10(eH0(t2)eH0t1). Consider a photon observed at t0. Find the distance at which it was emitted and write this in terms of the observed redshift z. Your answer should be an expression containing z, H0, and c. Vote this post up 0 Vote this post down + Add New Comment Sohan P Jain 1 week ago Someone, please clarify the expresion for D: Does H_0(t_2) stand for H_0*t_2 or H_0 as a function of t_2? Similar question for H_0t_1: Does it stand for H_0*t_1? (The two similar subexpressions are written differently.) Vote this post up 0 Vote this post down Comments Philip Tay (Student) 1 week ago Sohan, that pair of brackets around t_2 was unnecessary - see Ronan's thread above. I thought that was naughty of him but ok he admitted it. Vote this post up 0 Vote this post down + Add New Comment Colin Derek Dando 1 week ago All's well that ends well but that was confusing with so many ts. Vote this post up 0 Vote this post down + Add New Comment Tjerk Gauderis 1 week ago Ronen's extra question about the particle Horizon has really a mind blowing result... "hard to wrap my mind around it" Vote this post up 0 Vote this post down + Add New Comment Anonymous 3 days ago I wonder if someone can help me with this. I've come back to it time and time again and I'm still not getting it... conceptually or mathematically If I treat t2 as t and t1 as t0 I end up with eH0t1=1 and eH0t2=e1/(1+z)

I've read the post above and I think it's just that I don't understand what I'm aiming at here. Week 8 has been really tough! Vote this post up 0 Vote this post down Comments Anonymous 3 days ago Must be something about displaying silly mistakes in pubic that make them glaringly obvious!! Got it. Vote this post up 0 Vote this post down ronen plesserINSTRUCTOR 3 days ago Tell me about it...:)

Week 8, Part B, Question 7, Details


Sort replies by: Oldest first Newest first Most popular No tags yet. + Add Tag Anonymous 2 weeks ago Question 7 Assuming the value we measure for H0 find the distance to a source with z=5.34 (you can compare your answer to the results you found for a dust-filled universe in Part A). Express your answer in light-years and round to two significant figures. Vote this post up 0 Vote this post down Dimitris Nikolaidis 1 week ago Should the result be the same as the question 3 in part A ? Vote this post up 0 Vote this post down Comments A. 1 week ago It is true that both results are on the same order of magnitude but not exactly identical..Remember, that was a dust-dominated flat universe in Part A. Vote this post up 0 Vote this post down + Add New Comment Anonymous 1 week ago If you are having unit problems with this question, I can recommend going to the Q9 thread and picking up and using Stoica's H0conversion. Vote this post up 2 Vote this post down + Add New Comment Vlad Alexandru Niculescu 1 week ago huge lapsus here. Where is the formula for the distance in this case? i can't seem to locate it anywhere.. Vote this post up 0 Vote this post down Comments A. 1 week ago

It's just the answer to the previous QuB6. You found D=D(c,z,H0), remember? You are given z, you know c and H0, find Distance D expressed in lightyears. Careful on the unit conversions. Vote this post up 3 Vote this post down + Add New Comment Dimitris Nikolaidis 1 week ago This is the only question that I have not managed to find the correct answer. My answer on QB6 accepted by the A.G and the unit convertions are correct (I suppose). I had follow Stoica's convertions at QB9. Any ideas, what I can doing wrong? Thanks for your advice Vote this post up 0 Vote this post down Comments Ioanna 1 week ago Is your c/H0 of order e10 (in light years)? Vote this post up 0 Vote this post down A. 1 week ago Dimitri, If you cannot manage this one but you dealt with 8B3 with success, then something may be wrong with your units or your powers or both. Check again. Vote this post up 0 Vote this post down Anonymous 1 week ago Dimitri, Ioanna, Same thing here: all my other answers are accepted by the AG. Since H0 had to be used before, I suppose the conversion is fine. My c/Ho however is not in the order of e10 (in Ly) as Ioanna mentions? Since'"just" plugging in the data to my accepted answer of Q6 doesn't work. Do I oversimplify this thinking that just the equation of q6 is needed here..? I wonder what am I not grasping here? Any thoughts would be highly appreciated Vote this post up 0 Vote this post down Dimitris Nikolaidis 1 week ago No Ioanna, my results (all) are of order e11 ly Vote this post up 0 Vote this post down Dimitris Nikolaidis 1 week ago

I reached a result of the order of x.ye10 with x + y = 5 but the result by the AG was wrong again Vote this post up 0 Vote this post down + Add New Comment Ioanna 1 week ago Since you have qu6 correct, it means you are using the right formula. And it only involves, z, c, H0. So check your calculations: Giving rounded numbers as a guide, H0 = 2.4e-18 s^-1 1/H0= 4.3 e17 s 1/H0= 1.38 e10 yrs (remember this is the estimated age of our universe, and you know it is ~ 13.8 billion years old. So your 1/H0 must be around 1.38 e10 yrs, else something is wrong). Once you have it in years, you can use the approach that speed of light c, is 1 ly per year, so c/H0 should be around 13.8 billion light years. And you then just increase increase it by z. (you would need to use more accurate numbers than these that I mentioned as guide, else the the answer may be out of range of autograder). Vote this post up 0 Vote this post down Comments Dimitris Nikolaidis 1 week ago Ioanna, thanks a lot. Finally I managed this question, It was so simple, repeated wrong convertions that I can't see. Again thank you very much for your assistance. Vote this post up 0 Vote this post down Konstantinos Siettos 1 week ago How do we know that H_0=2.4e-18 s^-1? Vote this post up 0 Vote this post down Anonymous 1 week ago Konstantinos, the question states "Assuming the value we measure for H0" - this is a "real world" measurement established by measuring redshift in distant galaxies. The Wikipedia article on Hubble's Law has this to say "As of 20th Dec 2012 the Hubble constant, as measured by NASA's Wilkinson Microwave Anisotropy Probe (WMAP) and reported in arxiv (http://arxiv.org/pdf/1212.5225.pdf), is 69.32 0.80 (km/s)/Mpc".

To get from (km/s)/Mpc to s^-1, we take 69.32 and divide by the number of km in a Mpc, 3.09e19, which gives us 2.24e-18 s^-1 (slightly different from Ioanna's value, I think the Hubble Constant is given in the video lectures as something slightly different from the 69.32 found at Wikipedia). Vote this post up 2 Vote this post down + Add New Comment Anonymous 1 week ago Ioanna, Thought I found my error. (as we were warned in advance by ). Your last comment confirmed it.. And so does the AG. Thanks both of you for dragging me over the finish.. Best regards

Week 8, Part B, Question 8, Details


Sort replies by: Oldest first Newest first Most popular No tags yet. + Add Tag Anonymous 2 weeks ago Question 8 For a photon emitted at time t find the distance today of the most distant point it will ever reach. This is the event horizon at time t. Your answer should be an expression in terms of H0, c, and t. Vote this post up 0 Vote this post down Anonymous 2 weeks ago Shouldn't the answer also depend on t0? Vote this post up 0 Vote this post down + Add New Comment Anonymous 2 weeks ago In Clip 5, Slide 4, the horizon is given as proportional to a. If this formula is correct, then there may be dependence of t_0. Nevertheless, the grader refused my solution with t_0 and mentioned that it is a symbol not part of the problem. Vote this post up 0 Vote this post down + Add New Comment Nicholas 2 weeks ago I believe at the time (t_0) at which the photon will reach it's most distant point is consistent with t_0 approaching infinity. Vote this post up 1 Vote this post down + Add New Comment Anonymous 2 weeks ago According to Wiki, the event horizon is defined as: the event horizon is the largest comoving distance from which light emitted now can ever reach the observer in the future. So it involves integrating from t0 to infinity. This should give cH0, which of course is not accepted by the grader. Vote this post up -1 Vote this post down + Add New Comment
A post was deleted.

ronen plesserINSTRUCTOR 2 weeks ago In the exponential universe, H is a constant so H0 would be the same no matter when you measured it. As a result t0 will not appear in the answer to this one. The answer contains a factor of the form exp(x) but also a prefactor. Vote this post up 1 Vote this post down Comments Ioanna 2 weeks ago Thanks Ronen. It now works. I was using t (not t0), but I had a problem with my exponential term which I managed to fix (by guessing, mainly. If it not one then it is the other, sort of thinking). Vote this post up 0 Vote this post down Anonymous 2 weeks ago Prof. Ronen, I do not want to just make trial and error. What is the event horizon? Isn't it ctdta? Vote this post up 0 Vote this post down stephen gould 2 weeks ago I am sort of shocked but I just created an expression based on what we were given as simply as possible and it worked and then multiplied out an awful math thing for 9 and it worked as well. Now back to the question 5 but if Ioanna can't get it how can us mortals get it to work Oh and Ioanna thnaks - I don't usually get help from others but you were terrific and clearly have a good thought process Vote this post up 0 Vote this post down + Add New Comment A. 2 weeks ago If H0t0 appoaches infinity, then eH0t0 approaches 0. So, t0 is not included in the requested expression. Thank you guys for your insight. Vote this post up 2 Vote this post down + Add New Comment
A post was deleted.

Ioanna 2 weeks ago , I think in the thread for qu6, it was said t0=0 (so it does not approach infinity) Ok, I think we are still looking at the formula given in qu6:

D=cH10(eH0t2eH0t1)
t2 is the time of emission, and t1 is the later time. The formula gives us the distance D, that light emitted at t2, will reach at time t1 (the distance from where it was emitted).

So for this question we have that it is emitted at t (so t2=t). At some later time t1, it will be at distance D. What is the maximum distance it will reach? Well the more time passes the further away it goes, so max D is when t1+ So we use the above formula with t2=t (time of emission) and t1=+ (so t0 is not involved here at all) (That is not how I did it originally, I had no clue what to do and I somehow managed to solve qu9 first but constructing my own formula, which I then solved for D, and used it to answer qu8. Going completely backwards in other words ...) Vote this post up 21 Vote this post down Comments
A comment was deleted.

A. 2 weeks ago Well... thank you Ioanna! I only did this:

t1+t1H0t1eH0t1e=0 eH0t1=0eH0t1=0eH0teH0t1=eH0t c(eH0teH0t1)=ceH0t cH0(eH0teH0t1)=cH0eH0t,t<t1+


Both parts of the last equation have length dimensions. So, give me that time t of the photon emission and I will find you the distance today of the most distant point it will ever reach, namely the event horizon at time.. Sounds poetic, doesn't it? Vote this post up 8 Vote this post down Loh Siu Yin 2 weeks ago Thank you, Ioanna -- good and clear explanation! Vote this post up 0 Vote this post down Satish Pisharody (Student) 2 weeks ago A. , I get the same answer you found above, but the grader doesn't think it's poetic enough! May be, it's the way I've entered the answer in the grader that's the issue, especially the exponent part...or is it? Let me bang my head against the wall some more, it's bloody but unbowed.... Vote this post up 0 Vote this post down A. 1 week ago I almost give away the answer, so be careful on what you re entering in your submission. Careful with * and exp(x) stuff.. Vote this post up 0 Vote this post down

Judit 2 days ago Exp...? :S I tried e^( ) and it didn't work, I was reprimanded for including e at all :*( How do I enter "e to something" correctly? Vote this post up 0 Vote this post down
A comment was deleted.

Justin JohnsenSTAFF 2 days ago Sorry, wrong e. For the exponential function, use exp(x), where x is whatever you are putting as the power. So, exp(3) = e^3 Vote this post up 1 Vote this post down Judit 2 days ago Worked like a charm. Thanks! Vote this post up 0 Vote this post down Christoffel Johannes Lombard 16 hours ago So how do I enter infinity? Vote this post up 0 Vote this post down Christoffel Johannes Lombard 10 hours ago Please ignore my previous question. I have got the equation right, thanks for your help. I could not have done it without your posts. Vote this post up 0 Vote this post down + Add New Comment Anonymous 4 days ago Hello, in what form will the grader accept H_sub_0 to be written? Thanks! Julie Vote this post up 0 Vote this post down Comments Paul Robinson 4 days ago H underscore 0 Vote this post up 0 Vote this post down + Add New Comment

Bernard Colloff (Student) 11 hours ago Hello, Still having difficulty with this question. I would appreciate another review. Thanks for your help. Vote this post up 0 Vote this post down + Add New Comment serge 7 hours ago We have the working formula given in the Question 6 (the question itself, not the answer is meant) - the formula for distance D that the light emitted at time t1 reaches at the time t2. Note that this is NOT just c(t2t1). Why it is not so? Because the universe is expanding, and all possible means for measuring distance do so. You can check however, that for small times (small values H0t) this expression acquires more common form (use the calculus approximate formula exp(x)=1+x+ valid for x1 - surprisingly, it will turn to c(t2t1). So everything beyond is a cosmological correction depending upon the cosmological model accepted. The part A of this week related to the "dust-dominated" Universe with other dependence of distances traveled by light (but again which would reduce to common formula D=ct at small times). Now we have the model of exponentially expanding Universe which is supposedly the case of its later evolution from now on. This strange formula for distance is related to the scale dependence on time, a(t) which is also related to the cosmological model and can be found by solving Friedman equation by neglecting different terms in it. For dust dominated Universe we had a(t)=(afactor)t2/3, for the time t0 "now" it is 1. For exponentially expanding Universe it is given a(t)=exp(H0t); notice that at t=0 ("now") it is again 1; at times "before" with t<0 it is a number between 0 and 1; at times "after" with t>0 it is a number above 1. And third variant is given for the radiation era (W8, clip4, slide 4), where a(t)(t). But we are interested in exponential era now. The formula for the distance given above and in Q6 is just the integral ct2t1dta(t). If a were constant, this is just the statement, that the light for a small interval time t in some remote past travels as we perceive it from now not the distance ct, but this distance scaled by factor a: ct/a. For changing a(t) we need a calculus with this integral to arrive to the formula for D given here as ready-made. Anyhow, we have this formula for distance passed by light between times t1 and t2 - ALREADY corrected for cosmological expansion and as viewed from today. That is this is the distance which we and now measure for this light. Now let us think, what is the maximal distance the light ever travels if it is emitted at some time moment t. This is the time horizon. From outside this distance no event has chance to reach us and to be detected. So just take the formula for distance measured in given cosmology and consider what will be the maximal distance of light path for an event (flash of light, or, say, star explosion) occurred at time t. We can think of a partial case. Consider a light flash "now" at some distance from us. Will it be possible to see it at some time later? If it is close enough, the light from it will reach us after some time. If it is beyond that critical distance (time horizon) - the light will never reach us because the Universe at that point will be expanding faster than the light travels. This critical distance can be given from the formula if we set one time to "now", and other time - to very big value. And the general case is considered similarly, but instead of "now" we use a given time t.

Week 8, Part B, Question 9, Details


Sort replies by: Oldest first Newest first Most popular No tags yet. + Add Tag Anonymous 2 weeks ago Question 9 At what time t will this become of the order of the distance to the Andromeda galaxy (2.5 Mly)? Express your answer in years (counting from today) and round to two significant figures. Vote this post up 0 Vote this post down Jens-Peter Imohr 2 weeks ago for this one I could need some math-help. How to solve an equation like: exp(At)=B for t ? Vote this post up 2 Vote this post down + Add New Comment Shlomo Eshet 2 weeks ago LN(exp(-A*t)=LN(B) <=> -A*t=LN(B) <=> t=LN(B)/(-A) Vote this post up 5 Vote this post down Comments Jens-Peter Imohr 2 weeks ago Thanks a lot. I think now I can solve the last Question Vote this post up 0 Vote this post down Anonymous 1 week ago I have problem to understand LN? Please help explain. Vote this post up 0 Vote this post down A. 1 week ago ln(x) is the natural logarithm of x and is the inverse function of ex or exp(x) and vise versa. So, ln(exp(x))=ln(ex)=x and eln(x)=exp(ln(x))=x. So, if a is any known number and x is the unknown:

ex=aln(ex)=ln(a)x=ln(a) and you find x whatever that maybe, e.g. x = t, etc...


Vote this post up 4 Vote this post down

Joanna Loesch (Student) 1 week ago Thank you so much & Ioanna :) With your invaluable help I finally managed to wrestle down set B. I swear I feel 15cm taller now and I have the urge to go around, grab random people by lapels and shoult "I did it!" :D Vote this post up 3 Vote this post down + Add New Comment Satish Pisharody (Student) 2 weeks ago I think I'm messing up the units here a bit... I've converted the Distance (2.5 mill ly) from ly to parsec. H_0 is in kms/sec per megaparsec, which I've converted into kms/sec per parsec as well. c is in kms/sec. I land up with an expression like what Jens-Peter mentions exp(At)=B which then I'm trying to solve for t along the lines Shlomo has indicated.... I land up with a number (x.ye4, x+y = 10) I'm unable to figure the units of, and, of course, the grader doesn't like it either. What am I missing? I'm not too comfortable with Ln, so may be I'm making a mistake there. Or else in unit conversions. Help needed. Thanks! Vote this post up 0 Vote this post down Comments Lauren Suzanne Gonzalez 1 week ago I had that same thing (except my exponent was -4) with x+y=10, and it was dimensionless...due to the fact that H_0 can be converted to units related to 1/time, and it is multiplied by time. Vote this post up 0 Vote this post down Ioanna 1 week ago Satish, remember when you have exp(something), the something is unitless (so just a number). In our case it would be H0t, so (1/s)*s=1, no units, as it should be. When you take ln of something, the something is also unitless. Because lnx=y means 'to what exponent (x) do we need to raise e to get y'? so x is the exponent so unitless, so is its ln, and so is y. And ln[exp(something)]=something When dealing with ln and exp it is useful to remember we are dealing with ratios (and ratios are unitless, as long as they are in the same units so km with km, or pc with pc, but not say pc with km, or sec with years).

we are basically saying that a ratio of distances (so unitless, as long as we have converted both numerator and denominator to the same unit of length) is equal to exp(something), also unitless, and where the something is a ratio of times. Vote this post up 4 Vote this post down Satish Pisharody (Student) 1 week ago Thanks, Ioanna. You've been a great one to have around:) I sorted this one out a few hours back. It was, as I thought, a problem with unit conversions sometimes when in a hurry, one tends to do things mechanically.... now that I'm done with HW8 A&B and the course, want to go over all the math to see if I can derive a more deeper meaning to some of the HW problems (some, I admit I completed with only a shaky understanding of the underlying physics, at best). I suppose, the purpose of the HW (and indeed, of my taking up this course) is for us to gain a holistic understanding of the subject. It's in this area that these forums, with people like you in it, have been immensely helpful. Math might be great tool to understand & explore the physical sciences, but there are times I feel, too much math boggles the mind, makes the concepts far too abstract. Week 8 was one such, where, frankly, the conceptual understanding is trailing the HW completion by some distance! Hence, a little bit of 'mulling over' is in order, want to spend some time untangling the equations in my mind & understanding the underlying principles:) 't was great to have you guys in the "classroom", will sure bump into some of you in the other Coursera courses:) Vote this post up 0 Vote this post down A. 1 week ago Satish, agreeing you with you I would stress that I kinda liked more the "find the expression" questions rather than "you have km, now find it in ly, then find it in mm and then find it in Mpc.. Oh! Not in Mpc but in kpc!" questions. Sometimes I found myself struggling with unit conversions (God Save Excel) and in the meantime I lost all the underlying physics concepts. Found a number in kpc and I almost completely lost its meaning. I know I exaggerate a little and I am not saying that "gimme a number" questions were a bad thing but... ...on the other hand, "find the expression"questions focused me more on the symbols and the quantities they represented and what was their relation. What's that ? What's that R there? Hmm.. radius of a sphere! Well, may I know it's mass? May I know its volume? What t stands for in relation to t prime? I could present numerous similar examples. Those questions really stressed on the concepts and kept me really focused. And, most of all: no rounding errors, no marginal differences due to different procedures of calculation. Either you are ok and the grader says you're ok or you are not. One and only one expression could be true. Vote this post up 0 Vote this post down

Sohan P Jain 1 week ago I read the feedback on keeping track of units correctly. In case of HW8B/9, t has an expression that involves four subexpresssions ln(H_0), ln(2.5Mly), Ln(c) and H_0, all with different units. I had no problem resolving units without Ln function (for example, in case of HW8A/2 and HW8A/3). My first choice is to convert each of them to years as the answer is required in years. (Even ln(H_0) and H_0, logically, have different units. H_0 has units km/sec/MPC and ln(H_0) has units ln(km/sec/MPC). Isn't it? I tried different ways but I had no luck so far. Sometimes, I get negative value of t. I want to understand the correct logic instead of depending on autograder. (I have become so much dependent on autograder, I don't know if I will ever be sure about the accuracy of a solution without the autograder.) I am 100% sure my expression for t is correct. Vote this post up 0 Vote this post down + Add New Comment Lauren Suzanne Gonzalez 1 week ago I got a number ~e6, which seems reasonable, given the distance of 2.5MLY, but my answer was not accepted. Can anyone confirm a similar answer? There are so many conversions in this problem, and so many places to make mistakes. Vote this post up 0 Vote this post down Comments Lauren Suzanne Gonzalez 1 week ago Oops, pc to mpc and a mis-typed number changed my order of magnitude from 6 to 8. Still wrong. Ideas? Vote this post up 0 Vote this post down + Add New Comment Paul Robinson 1 week ago Lauren, your order of magnitude is still too small. My approach was to convert all the inputs into km for distance and seconds for time. Then at the end convert my seconds into years. Vote this post up 0 Vote this post down Comments Lauren Suzanne Gonzalez 1 week ago Ah, that did it! Thanks so much!! Vote this post up 0 Vote this post down

Paul Robinson 1 week ago Glad to give a little back, after getting so much help along the way from fellow students :-) Vote this post up 1 Vote this post down + Add New Comment Paul Robinson 1 week ago Dimensional analysis:

cH10e(H0t)=D Since H0 has units of s1, H10 has units of s kmsse(s1s)=D km=D
Vote this post up 0 Vote this post down Comments Satish Pisharody (Student) 1 week ago Thanks, Paul. Had tied myself up in knots with all the unit conversions. Your dimensional analysis brought some sanity into it:) Vote this post up 0 Vote this post down Sohan P Jain 1 week ago But H_0 has units km/s/MPC. Isn't it? Vote this post up 0 Vote this post down Paul Robinson 1 week ago H_0 is sometimes expressed in km/s per Mpc as an easy to use number. This is distance/time * 1/distance = 1/time Vote this post up 0 Vote this post down
A comment was deleted.

+ Add New Comment Stoica Dorian - Bogdan 1 week ago I expressed H_0 in seconds, and then in years. . Ugly numbers, i

know, but because we have the distance in Mly, we can set c=1 lightyear / year, and the result will be in years. Vote this post up 10 Vote this post down Comments Anonymous 1 week ago Excellent - very useful here and in Q7 Vote this post up 0 Vote this post down Oscar Orta (Student) 1 week ago Stoica: That idea was great....thank you for it.Helped a lot to solve the equation correctly. Vote this post up 0 Vote this post down Anonymous 1 week ago I have problem to understand "ln" and to solve exp (-A*t) = B for t? Please help. Vote this post up 0 Vote this post down Wheeler Huneycutt 1 week ago Dudes: Type this into search box: "71 km/s = ? Mpc / yr" Vote this post up 1 Vote this post down Sohan P Jain 1 week ago Stocia, Thank you. It works! Vote this post up 0 Vote this post down MALINA ELEONORA COCIOCEANU 3 days ago Thank you, Bogdan.Very useful your post! Vote this post up 0 Vote this post down SANTIAGO ARVALO GONZLEZ 1 day ago No me da! La respuesta me da en orden de E12 o E13 pero no la acepta el autograder Vote this post up 0 Vote this post down Judit 1 day ago

Thanks Stoica, your insight regarding c helped a lot. Vote this post up 0 Vote this post down + Add New Comment
A post was deleted.

Yandi Jaya 1 week ago Holy stars and garters, I'm done. I'M DONE! Vote this post up 3 Vote this post down + Add New Comment Stoica Dorian - Bogdan 1 week ago

1. Anon, is equivalent with 2. The general rule is: if then 3. Example 1: if then

. So . .

4. Example 2: if then 5. This should be useful: http://en.wikipedia.org/wiki/Natural_logarithm Vote this post up 1 Vote this post down

+ Add New Comment Wheeler Huneycutt 1 week ago Dudes!! and Dudettes!!! Veni Vedi Fini !!!! Vote this post up 4 Vote this post down + Add New Comment Nirmala Sonathi (Student) 1 week ago oops! This one is taking me on a joy-ride.I am getting xye9, where x+y=7. 2.5Mly=2.5* 10^6* 9.460536207e12 km. I use c=2.99792458e5 km/s, value of H_0=2.30095e-18/s. I get my answer in sec which i convert to years. Where am i doing wrong? This last one seems easy but consuming all my time.Can someone help. Vote this post up 0 Vote this post down Comments Anonymous 1 week ago Stoica, thanks for your advices on the links, they are truly helpful.

Vote this post up 0 Vote this post down derek young 1 week ago I too am using the same values ... c=2.9979e5 km/s H_0=2.30095e-18 /s D=2.365e19 km equation from qu8 becomes (log((D*H_0)/c))/H_0 and answer is x.ye10 to 2sig figs ... (x+y=7) ... which is wrong ... Driving me nuts ... Where am I going wrong? Vote this post up 0 Vote this post down Samarth Agarwal 1 week ago i think it should be ln and not log. (here 'e' is 2.71 not our 10^x thing!!) hope u get the right answer then :) Vote this post up 0 Vote this post down derek young 1 week ago aaaaaaaaargh. /facepalm. Vote this post up 0 Vote this post down Nirmala Sonathi (Student) 1 week ago Thanks Samarth.Actually i was also doing log. Chalo now i can move on to my next course. Vote this post up 0 Vote this post down Samarth Agarwal 1 week ago :) Vote this post up 0 Vote this post down + Add New Comment Colin Derek Dando 1 week ago Derek, I have the same units as you but my quotient is inverted because of the -ve power (exponent) from Q8. Hence (ln(c/D*H_0))/H_0. What do you think? Vote this post up 0 Vote this post down + Add New Comment Pierre FERNANDEZ 1 week ago

Bravo to Ronen and his staff, and to all those who communicated on the forums. Thanks to you, I learnt, understood, and responded with success to all the HW (except two or three questions where I mixed the units! Shame on me!). The chapter 8 concerning the cosmology is excellent. I ignored everything on the scale factor and other elements presented by Ronen. But the subject is difficult, and I did not really assimilate all the concepts developed in this section. For example, I am very surprised by my result in this question 8B9, (near 100 times the age of the universe) and I really doubted before subjecting it to the autograder ! The future belongs to photons ! Vote this post up 0 Vote this post down + Add New Comment Richard F Riccelli 3 days ago I am sad to see this adventure end! Sincere thanks to Ronen and Justin for all their effort in making this such an engaging and rewarding experience. Also thanks to Ioanna, (aka Greek Guy), Anonymous (you know who you are) and all the various forum contributors who have helped me and others find the way when needed. And let's not forget the auto-grader... :-) I hope to see many of you again at Professor Ronen's next opus on Coursera. (hint!) Rick R. Vote this post up 6 Vote this post down + Add New Comment Anna Czarina V. Cabe 1 day ago Yay! Finaly! Thank you very much Stoica! And this ends IntroAstro Homeworks! Vote this post up 0 Vote this post down + Add New Comment Judit 1 day ago I have the following prob. I get a positive nr for ln(B). Shouldn't that be negative in order to get a positive t when divided by -H0? Vote this post up 0 Vote this post down Comments Anna Czarina V. Cabe 1 day ago Stoica's post above should save you from a lot of conversion factors, and it makes c=1, so the resulting expression is not too messy. Vote this post up 0 Vote this post down Anna Czarina V. Cabe 1 day ago

Judit, yes, ln(B) should be negative. Vote this post up 0 Vote this post down + Add New Comment Wheeler Huneycutt 1 day ago Judit, You are correct , use -H0 but make sure H0 is in 1/year terms both for taking the ln on top as well as bottom term. and put c in ly/year too. Vote this post up 1 Vote this post down Comments Judit 1 day ago Thanks! Indeed, I messed up something around H0, I got some weird exponents there. Also, it helps if one doesn't feed 25 Mly into Excel as 2,5*10^6.... Unit conversions are the root of all evil. And commas. And periods. And late night calculations. I was finally able to sort out my numbers and get the correct answer. Phew. Vote this post up 0 Vote this post down + Add New Comment Jim Olson 1 day ago Finished finally! So long and thanks for all the hints... Vote this post up 2 Vote this post down + Add New Comment GERARDO IVAN VELAZQUEZ ALVARADO (Student) 23 hours ago So, this is it, thanks to Professor Ronen Thanks to Ioanna, , Wheeler, and all of the crue, see you again when it happens, so be it!

Você também pode gostar